Вы находитесь на странице: 1из 16

Department of Mathematics

Indian Institute of Technology Delhi


MAL 110: Calculus
Lecture Notes
Contents
Sequences of real numbers: Denitions of sequence and convergence, subsequences, bounded
sequences, operations on sequences, monotone sequences,Cauchy sequences, Limit supe-
rior and inferior, divergent sequences, examples of some special sequences.
1
MAL 110: Calculus
Lecture Notes
1 Sequences of real numbers
1.1 Sequences and their limit
Denition 1.1.1. A sequence of real numbers is a function from the set of positive integers
to R.
Notation. It is customary to denote a sequence as {a
n
}

n=1
.
Examples 1.1.2. (i) {c}

n=1
, c R, (ii) {
(1)
n+1
n
}

n=1
, (iii) {
n1
n
}

n=1
and (iv) {

n}

n=1
.
Denition 1.1.3. A sequence {a
n
}

n=1
converges to limit L if for every > 0 (given)
there exists a positive integer N such that n N = |a
n
L| < .
Notation. L = lim
n
a
n
or a
n
L.
Examples 1.1.4.
(i) It is clear that the constant sequence {c}

n=1
, c R, has c as its limit.
(ii) Show that lim
n
1
n
= 0.
Solution. Let > 0 be given. In order to show that 1/n approaches 0, we must
show that there exists an integer N N such that for all n N,

1
n
0

=
1
n
< .
But 1/n < n > 1/. Thus, if we choose N N such that N > 1/, then for all
n N, 1/n < .
(iii) Consider the sequence {(1)
n+1
}

n=1
. It is intuitively clear that this sequence does
not have a limit point or it does not approach to any real number. We now prove
this by denition. Assume to the contrary, that there exists an L R such that the
sequence {(1)
n+1
}

n=1
converges to L. Then for =
1
2
, there exists an N N such
that
|(1)
n+1
L| <
1
2
, n N. (1.1)
For n even, (1.1) says
| 1 L| <
1
2
, n N. (1.2)
2
while for n odd, (1.1) says
|1 L| <
1
2
, n N. (1.3)
which is a contradiction as 2 = |1 + 1| |1 L| +|1 + L| < 1.
Lemma 1.1.5. If {a
n
}

1
is a sequence and if both lim
n
a
n
= L and lim
n
a
n
= M holds,
then L = M.
Proof. Suppose that L = M. Then |L M| > 0. Let =
|LM|
2
. As lim
n
a
n
= L, there
exists N
1
N such that |a
n
L| < for all n N
1
. Also as lim
n
a
n
= M,there exists
N
2
N such that |a
n
M| < for all n N
2
. Let N = max{N
1
, N
2
}. Then for all
n N, |a
n
L| < and |a
n
M| < . Thus |LM| |a
n
L| +|a
n
M| < 2 = |LM|,
which is a contradiction. ///
Theorem 1.1.6 (Sandwich theorem for sequences). Let {a
n
}, {b
n
} and {c
n
} be three
sequences such that a
n
b
n
c
n
for all n N. If lim
n
a
n
= L and lim
n
c
n
= L, then
lim
n
b
n
= L.
Proof. Let > 0 be given. As lim
n
a
n
= L, there exists N
1
N such that
n N
1
= |a
n
L| < . (1.4)
Similarly as lim
n
c
n
= L, there exists N
2
N
n N
2
= |c
n
L| < . (1.5)
Let N = max{N
1
, N
2
}. Then, L < a
n
(from (1.4)) and c
n
L+ ( from (1.5)). Thus
L < a
n
b
n
c
n
L + .
Thus |b
n
L| < for all n N. Hence the proof. ///
Examples 1.1.7.
(i) Consider the sequence
_
cos n
n
_

n=1
. Then
1
n

cos n
n

1
n
. Hence by Sandwich
theorem lim
n
cos n
n
= 0.
(ii) As 0
1
2
n

1
n
and
1
n
0 as n ,
1
2
n
also converges to 0 by Sandwich theorem.
(iii) (1)
n
1
n
0 as
1
n
(1)
n
1
n

1
n
for all n 1.
3
(iv) If b > 1, then lim
n
n

b = 1.
Solution. Let a
n
= b
1
n
1. As b > 1, a
n
0 for all n N. Further,
b = (1 + a
n
)
n
=
n

k=0
n
C
k
a
k
n
1 + na
n
.
Then 0 a
n

b 1
n
. Thus a
n
0, i.e., b
1
n
1 as n . ///
Denition 1.1.8. (Subsequence): Let {a
n
} be a sequence and {n
1
, n
2
, ...} be a sequence
of positive integers such that i > j implies n
i
> n
j
. Then the sequence {a
n
i
}

i=1
is called
a subsequence of {a
n
}.
Theorem 1.1.9. If the sequence of real numbers {a
n
}

1
, is convergent to L, then any
subsequence of {a
n
} is also convergent to L.
Proof. Let {n
i
}

i=1
be a sequence of positive integers such that {a
n
i
}

i=1
forms a subse-
quence of {a
n
}. Let > 0 be given. As {a
n
} converges to L, there exists N N such
that
|a
n
L| < , n N.
Choose M N such that n
i
N for i M. Then
|a
n
i
L| < , i M.
Hence the proof. ///
Denition 1.1.10. (Bounded sequence): A sequence {a
n
} is said to be bounded above,
if there exists M R such that a
n
M for all n N. Similarly, we say that a sequence
{a
n
} is bounded below, if there exists N R such that a
n
N for all n N. Thus a
sequence {a
n
} is said to be bounded if it is both bounded above and below.
Lemma 1.1.11. Every convergent sequence is bounded.
Proof. Let {a
n
} be a convergent sequence and L = lim
n
a
n
. Let = 1. Then there exists
N N such that |a
n
L| < 1 for all n N. Further,
|a
n
| = |a
n
L + L| |a
n
L| + L < 1 + L, n N.
Let M = max{|a
1
|, |a
2
|, ..., |a
n1
|, 1 + |L|}. Then |a
n
| M for all n N. Hence {a
n
} is
bounded. ///
4
1.2 Operations on convergent sequences
Theorem 1.2.1. Let {a
n
}

1
and {b
n
}

1
be two sequences such that lim
n
a
n
= L and
lim
n
b
n
= M. Then
(i) lim
n
(a
n
+ b
n
) = L + M.
(ii) lim
n
(ca
n
) = cL, c R..
(iii) lim
n
(a
n
b
n
) = LM.
(iv) lim
n
_
a
n
b
n
_
=
L
M
if M = 0.
Proof. (i) Let > 0. Since a
n
converges to L there exists N
1
N such that
|a
n
L| < /2 n N
1
.
Also, as b
n
converges to M there exists N
2
N such that
|b
n
M| < /2 n N
2
.
Thus
|(a
n
+ b
n
) (L + M)| |a
n
L| +|b
N
M| < n N = max{N
1
.N
2
}.
(ii) Easy to prove. Hence left as an exercise to the students.
(iii) Let > 0. Since a
n
is a convergent sequence, it is bounded by M (say). Also as a
n
converges to L there exists N
1
N such that
|a
n
L| < /M n N
1
.
Similarly as b
n
converges to M there exists N
2
N such that
|b
n
M| < /M n N
2
.
Let N = max{N
1
, N
2
}. Then
|a
n
b
n
LM| = |a
n
b
n
a
n
M + a
n
M LM| |a
n
(b
n
M)| +|M(a
n
L)|
= |a
n
||b
n
M| +|M||a
n
L| < /2 + /2 =
5
(iv) In order to prove this, it is enough to prove that if lim
n
a
n
= L, L = 0, then
lim
n
1/a
n
= 1/L. Without loss of generality, let us assume that M > 0. Let
> 0 be given. As {a
n
} forms a convergent sequence, it is bounded. Choose N
1
N
such that a
n
> L/2 for all n N
1
. Also, as a
n
converges to L, there exists N
2
N
such that |a
n
L| < L
2
/2 for all n N
2
. Let N = max{N
1
, N
2
}. Then
n N =

1
a
n

1
L

=
|a
n
L|
|a
n
L|
<
2
L
2
L
2

2
= . ///
Examples 1.2.2.
(i) Consider the sequence
_
5
n
2
_

1
. Then lim
n
5
n
2
= lim
n
5
1
n

1
n
= 5 lim
n
1
n
lim
n
1
n
=
5 0 0 = 0.
(ii) Consider the sequence
_
3n
2
6n
5n
2
+ 4
_

1
. Notice that
3n
2
6n
5n
2
+ 4
=
3 6/n
5 + 4/n
.
Thus
lim
n
3n
2
6n
5n
2
+ 4
= lim
n
3 6/n
5 + 4/n
=
3 lim
6
n
5 + lim
n
4
n
= 3/5.
(iii) Similarly one can show that lim
n
_
n 1
n
_
= 1.
1.3 Monotone sequence
Denition 1.3.1. A sequence {a
n
} of real numbers is called a nondecreasing sequence if
a
n
a
n+1
for all n N and {a
n
} is called a nonincreasing sequence if a
n
a
n+1
for all
n N. A sequence that is nondecreasing or nonincreasing is called a monotone sequence.
Examples 1.3.2.
(i) The sequences {1 1/n}, {n
3
} are nondecreasing sequences.
(ii) The sequences {1/n}, {1/n
2
} are nonincreasing sequences.
(iii) The sequences {(1)
n
}, {cos(
n
3
)}, {(1)
n
n}, {
(1)
n
n
} and {n
1/n
} are not monotonic
sequences.
Theorem 1.3.3.
(i) A nondecreasing sequence which is bounded above is convergent.
6
(ii) A nonincreasing sequence which is bounded below is convergent.
Proof. (i) Let {a
n
}be a nondecreasing, bounded above sequence and a = sup
nN
a
n
. Since
the sequence is bounded, a R. We claim that a is the limit point of the sequence {a
n
}.
Indeed, let > 0 be given. Since a is not an upper bound for {a
n
}, there exists N N
such that a
N
> a . As the sequence is nondecreasing, we have a < a
N
a
n
for all
n N. Also it is clear that a
n
a for all n N. Thus,
a a
n
a + , n N.
Hence the proof.
The proof of (ii) is similar to (i) and is left as an exercise to the students. ///
Examples 1.3.4.
(i) If 0 < b < 1, then the sequence {b
n
}

1
converges to 0.
Solution. We may write b
n+1
= b
n
b < b
n
. Hence {b
n
} is nonincreasing. Since
b
n
> 0 for all n N, the sequence {b
n
} is bounded below. Hence, by the above
theorem, {b
n
} converges. Let L = lim
n
b
n
. Further, lim
n
b
n+1
= lim
n
b b
n
=
b lim
n
b
n
= b L. Thus the sequence {b
n+1
} converges to b L. On the otherhand,
{b
n+1
} is a subsequence of {b
n
}. Hence L = b L which implies L = 0 as b = 1.
(ii) The sequence {(1 + 1/n)
n
}

1
is convergent.
Solution. Let a
n
= (1 + 1/n)
n
=
n

k=0
_
n
k
__
1
n
_
k
. For k = 1, 2, ..., n, the (k + 1)
th
term in the expansion is
n(n 1)(n 2) (n k + 1)
1 2 k
1
n
k
=
1
k!
_
1
1
n
__
1
2
n
_

_
1
k 1
n
_
. (1.6)
Similarly, if we expand a
n+1
, then we obtain (n +2) terms in the expansion and for
n = 1, 2, 3, ..., the (k + 1)
th
term is
1
k!
_
1
1
n + 1
__
1
2
n + 1
_

_
1
k 1
n + 1
_
. (1.7)
It is clear that (1.7) is greater than or equal to (1.6) and hence a
n
a
n+1
which
7
implies that {a
n
}is nondecreasing. Further,
a
n
=(1 + 1/n)
n
=
n

k=0
_
n
k
__
1
n
_
k
< 2 +
n

k=2
1
k!
< 2 +
n

k=2
1
2
k
=2 +
(1/2) (1/2)
n
1 1/2
< 3.
Thus {a
n
}is a bounded monotone sequence and hence convergent.
Theorem 1.3.5. Every sequence has a monotone subsequence.
Proof. Pick x
N
1
such that x
n
x
N
1
for all n > N
1
. We call such x
N
as peak. If we
are able to pick innitely many x

N
i
s, then {x
N
i
} is decreasing and we are done. If there
are only nitely many x

N
s and let x
n
1
be the last peak. Then we can choose n
2
such
that x
n
2
x
n
1
. Again x
n
2
is not a peak. So we can choose x
n
3
such that x
n
3
x
n
2
.
Proceeding this way, we get a non-decreasing sub-sequence.
The following theorem is Bolzano-Weierstrass theorem. Proof is a consequence of Theorem1.3.5
Theorem 1.3.6. Every bounded sequence has a convergent subsequence.
1.4 Cauchy sequence
Denition 1.4.1. A sequence {a
n
}is called a Cauchy sequence if for any given > 0,
there exists N N such that n, m N = |a
n
a
m
| < .
Example 1.4.2. Let {a
n
}be a sequence such that {a
n
}converges to L (say). Let > 0 be
given. Then there exists N N such that
|a
n
L| <

2
n N.
Thus if n, m N, we have
|a
n
a
m
| |a
n
L| +|a
m
L| <

2
+

2
= .
Thus {a
n
}is Cauchy.
Lemma 1.4.3. If {a
n
}is a Cauchy sequence, then {a
n
}is bounded.
Proof. Since {a
n
}forms a Cauchy sequence, for = 1 there exists N N such that
|a
n
a
m
| < 1, n, m N.
8
In particular,
|a
n
a
N
| < 1, n N.
Hence if n N, then
|a
n
| |a
n
a
N
| +|a
N
| < 1 +|a
N
|, n N.
Let M = max{|a
1
|, |a
2
|, ..., |a
N1
|, 1 + |a
N
|}. Then |a
n
| M for all n N. Hence {a
n
}is
bounded. ///
Theorem 1.4.4. If {a
n
}is a Cauchy sequence, then {a
n
}is convergent.
Proof. Let a
n
k
be a monotone subsequence of the Cauchy sequence {a
n
}. Then a
n
k
is a
bounded, monotone subsequence. Hence {a
n
k
} converges to L(say). Now we claim that
the sequence {a
n
} itself converges to L. Let > 0. Choose N
1
, N
2
such that
n, n
k
N
1
= |a
n
a
n
k
| < /2
n
k
N
2
= |a
n
k
a| < /2.
Then
n, n
k
max{N, N
1
} = |a
n
a| |a
n
a
n
k
| +a
n
k
a| < .
Hence the claim. ///
Problem: Let {a
n
}de dened as a
1
= 1, a
n+1
= 1 +
1
an
. The show that {a
n
}is Cauchy.
Solution: Note that a
n
> 1 and a
n
a
n1
= a
n1
+ 1 > 2. Then
|a
n+1
a
n
| = |
a
n1
a
n
a
n
a
n1
|
1
2
|a
n
a
n1
|
1
2
n1
|a
2
a
1
|, n 2.
Hence
|a
m
a
n
| |a
m
a
m1
| +|a
m1
a
m2
| + .... +|a
n+1
a
n
| |a
2
a
1
|

n1
1
, =
1
2
So given, > 0, we can choose N such that
1
2
N1
<

2
.
1.5 Limit superior and limit inferior
Denition 1.5.1. Let {a
n
}be a bounded sequence. Then limit superior of the sequence
{a
n
}, denoted by limsup
n
a
n
, is dened as
limsup
n
a
n
:= inf
kN
sup
nk
a
n
.
9
Similarly limit inferior of the sequence {a
n
}, denoted by liminf
n
a
n
, is dened as
liminf
n
a
n
:= sup
kN
inf
nk
a
n
.
Example 1.5.2. (i) Consider the sequence {a
n
} = {0, 1, 0, 1, .....}. Then
n
= sup{a
m
, m
n} = 1 and
n
= inf{a
m
, m n} = 0. Therefore, liminf a
n
= 0, limsup a
n
= 1.
(ii) Consider the sequence {a
n
} = {
1
2
,
2
3
,
1
3
,
3
4
, ....}. Then for large k
1 sup{a
m
, m k} lim
k 1
k
0 < inf{a
m
, m k} lim
1
k
Then by sandwich theorem, we see that limsup a
n
= 1 and liminf a
n
= 0.
Lemma 1.5.3.
(i) If {a
n
}is a bounded sequence, then limsup
n
a
n
liminf
n
a
n
.
(ii) If {a
n
}and {b
n
} are bounded sequences of real numbers and if a
n
b
n
for all n N,
then
limsup
n
a
n
limsup
n
b
n
and
liminf
n
a
n
liminf
n
b
n
.
(iii) Let {a
n
}and {b
n
} are bounded sequences of real numbers. Then
limsup
n
(a
n
+ b
n
) limsup
n
a
n
+ limsup
n
b
n
and
liminf
n
(a
n
+ b
n
) liminf
n
a
n
+ liminf
n
b
n
.
Example 1.5.4. Consider the sequences {(1)
n
} and {(1)
n+1
}. Here a
n
= (1)
n
and
b
n
= (1)
n+1
. Also limsup
n
a
n
= limsup
n
b
n
= 1. But a
n
+b
n
= 0 for all n N and hence
limsup
n
(a
n
+ b
n
) = 0. Thus a strict inequality may hold in (iii) the above Lemma.
Theorem 1.5.5. If {a
n
}is a convergent sequence, then
liminf
n
a
n
= lim
n
a
n
= limsup
n
a
n
.
10
Proof. Let L = lim
n
a
n
. Then given > 0 there exists N N such that
|a
n
L| < , n N.
Equivalently L < a
n
< L+, for all n N. Thus, if n N, L+ is an upper bound
for the set {a
k
|k N}. If
k
:= sup{a
k
|k n}, then we note that L <
N
L+ and

N+1
< L + , ....,
n
< L + for all n N (As
n
is decreasing). Also a
n
> L , n
N =
n
L, n N. Therefore, lim
n
= L. Hence limsup a
n
= L. Similarly, one
can prove the liminf a
n
= L.
///
Theorem 1.5.6. If {a
n
}is a bounded sequence and if limsup
n
a
n
= liminf
n
a
n
= L, L R,
then {a
n
}is a convergent sequence.
Proof. Notice that
limsup
n
a
n
= lim
n
(sup{a
k
|k n})
and
liminf
n
a
n
= lim
n
(inf{a
k
|k n}).
Given that L = lim sup
n
a
n
. Thus for > 0, there exists N
1
N such that
| sup{a
n
, a
n+1
, ...} L| < , n N
1
.
This implies
a
n
< L + , n N
1
(1.8)
Similarly there exists N
2
N such that
| inf{a
n
, a
n+1
, ...} L| < , n N
2
.
This implies
L < a
n
, n N
2
(1.9)
Let N = max{N
1
, N
2
}. Then from (1.8)and (1.9) we get
|a
n
L| < , n N.
Thus the sequence {a
n
}converges. ///
Examples 1.5.7. lim
n
_
1 +
1
n
_
n
= e. Assume that e = lim
n

n
k=0
1
k!
.
11
Solution. Let a
n
=
n

k=0
1
k!
and b
n
=
_
1 +
1
n
_
n
. Now,
b
n
=
n

k=0
n
C
k
_
1
n
_
k
= 2 +
n

k=2
1
k!
k1

i=1
_
1
i
n
_
a
n
.(see (1.6)
This implies
limsup
n
b
n
limsup
n
a
n
= e.
Further, if n m, then
b
n
=
_
1 +
1
n
_
n
=
n

k=0
n
C
k
_
1
n
_
k

k=0
n
C
k
_
1
n
_
k
= 2 +
m

k=2
1
k!
k1

i=1
_
1
i
n
_
.
Keeping m xed and letting n , we get
liminf
n
b
n

m

k=0
1
k!
which implies a
n
liminf
n
b
n
. Hence
e = liminf
n
a
n
liminf
n
b
n
.
Thus
liminf
n
b
n
= limsup
n
b
n
= e,
1.6 Divergent sequence
Denition 1.6.1. Let {a
n
}be a sequence of real numbers. We say that a
n
approaches
innity or diverges to innity, if for any real number M > 0, there is a positive integer
N such that
n N = a
n
M.
If a
n
approaches innity, then we write a
n
as n .
A similar denition is given for the sequences diverging to . In this case we
write a
n
as n .
Examples 1.6.2.
(i) The sequence {log(1/n)}

1
diverges to . In order to prove this, for any M > 0,
12
we must produce a N N such that
log(1/n) < M, n N.
But this is equivalent to saying that n > e
M
, n N. Choose N e
M
. Then, for
this choice of N,
log(1/n) < M, n N.
Thus {log(1/n)}

1
diverges to .
(ii) Consider the sequence {(1)
n+1
n}

1
. Although this is not a convergent sequence, it
also does not approach to or .
(iii) The sequence (1)
n
is also an example of the previous type.
Denition 1.6.3. If a sequence {a
n
}does not converge to a value in R and also does not
diverge to or , we say that {a
n
}oscillates.
Lemma 1.6.4.
(i) A nondecreasing sequence which is not bounded above diverges to .
(ii) A nonincreasing sequence which is not bounded above diverges to .
Example 1.6.5. If b > 1, then the sequence {b
n
}

1
diverges to .
Lemma 1.6.6. Let {a
n
}and {b
n
} be two sequences.
(i) If {a
n
}and {b
n
} both diverges to , then the sequences {a
n
+ b
n
} and {a
n
b
n
} also
diverges to .
(ii) If {a
n
}diverges to and {b
n
} converges then {a
n
+ b
n
} diverges to .
Example 1.6.7. Consider the sequence {

n + 1

n}

n=1
. We know that

n + 1 and

n both converges to . But the sequence {

n + 1

n}

n=1
converges to 0. To see this,
notice that, for a given > 0,

n + 1

n < if and only if 1 <


2
+ 2

n. Thus, if N
is such that N >
1
4
2
, then for all n N,

n + 1

n < . Thus

n + 1

n converges
to 0. This example shows that the sequence formed by taking dierence of two diverging
sequences may converge.
1.7 Some special sequences
Theorem 1.7.1.
(i) lim
n
n

n = 1.
13
(ii) If x > 0 then lim
n
n
x
(1+x)
n
= 0.
(iii) If p > 0, then lim
n
log(n)
n
p
= 0.
Proof. (i) Let a
n
= n
1
n
1. Then 0 a
n
1 for all n N. Further,
n = (1 + a
n
)
n
=
n

k=0
n
C
k
a
k
n

n
C
2
a
2
n
=
n(n 1)
2
a
2
n
.
Thus 0 a
n

_
2
(n1)
(n 2). As
_
2
(n1)
0 as n , by Sandwich theorem,
a
n
0, i.e., n
1
n
1 as n .
(ii) Let k be an integer such that k > x, k > 0. Then for n > 2k,
(1 + p)
n
>
n
C
k
p
k
=
n!
k!(n k)!
p
k
=
p
k
k!
k

i=1
[n i + 1] >
n
k
2
k
p
k
k!
.
Hence,
0 <
n
x
(1 + x)
n
<
2
k
k!
x
k
n
xk
(n > 2k).
As x k < 0, n
xk
0. Thus
n
x
(1 + x)
n
0 as n .
(iii) For any n N there exists m N such that m n
p
< (m + 1) or equivalently
m
1
p
n < (m + 1)
1
p
. Let > 0. Since n
1
n
1 as n , there exists N N such that
n
1
n
(e

, e

), n N (or)
log n
n
(, ), n N. That is
log n
n
0 as n . This
implies that
lim
m
1
p
log(m + 1)
m
= 0.
As
1
n
p
<
log n
n
p
<
1
p
log(m + 1)
m
. Now the conclusion follows from Sandwich theorem. ///
1.8 Problems
1. For p > 0, show that lim
n
1
n
p
= 0.
2. Let a, a
1
R. For n 1, the sequence a
n
is dened by 2a
n+1
+ a
n
= a. Show that
the sequence converges and its limit is
a
3
.
3. Show that the sequence
_
n

i=1
1

n
2
+i
_
converges to 1.
4. If {a
n
} is a sequence of real numbers and if the subsequences {a
2n
}

n=1
and {a
2n1
}

n=1
both converge to the same limit, then show that a
n
also converges to the same limit.
14
5. Prove that if lim
n
an
n
= a = 0, then {a
n
} is not bounded.
6. If {a
n
} is a bounded sequence and {b
n
} is another sequence which converges to 0,
show that the product sequence also converges to 0. What can you say about the
product sequence, if {b
n
} converges, but to a non-zero point?
7. Let {a
n
} be a sequence of real numbers. Dene the sequence {s
n
} by s
n
=
1
n
n

i=1
a
i
.
(i) If {a
n
} is monotone and bounded show that {s
n
} is also monotone and bounded.
(ii) If {a
n
} converges to a, then show that the sequence {s
n
} also converges to a.
8. Let a
n
=
10
n
n!
. Find N N such that n N = a
n+1
< a
n
.
9. If a
1
>

2 and for n 2, a
n2
=
1
2
_
a
n
+
2
an
_
, then show that the sequence {a
n
}
is nonincreasing and bounded. Also, nd the limit.
10. Find the limit superior and the limit inferior for the sequence {(1)
n
_
1 +
1
n
_
}

n=1
.
11. (Interest rate models): If you invest an amount of money A
0
at a xed annual
interest rate r compounded m times per year, and if the constant amount b is added
to the account at the end of each compounding period (or taken from account if
b < 0), then the amount you have after n + 1 compounding period is
A
n+1
= (1 +
r
m
)A
n
+ b
(a) If A
0
= 1000, r = 0.02015, m = 12, and b = 50. How much money is in your
account at the end of 5 years? Does {A
n
} converge? Is {A
n
} bounded?
(b) Repeat part (a) with A
0
= 5000, r = 0.0589, m = 12 and b = 50.
(c) If you invest Rs.5000 in a deposit that pays 4.5/100 annually, compounded
quarterly and you make no further investments, how many years will it take
before you have Rs.20000? What if deposit earns 6.5/100?
12. (Dierence equations & Economics): Consider the model of closed economy. Let Y
denote the output, C denote connsumption expenditure and I denote investiment
expenditure. The superscripts n and n 1 denote the respective time periods:
Y
n
= C
n
+ I
n
, C
n
= 200 +
3
4
Y
n1
, I
n
= 150 + 0.15Y
n1
, Y
0
= 4000. Solve the
system and show that {Y
n
} converges as time n .
13. (Second order dierence equations): Consider the second order dierence equation
u
n+2
= au
n+1
+bu
n
, where a, b are constants. Assuming that u
n
= r
n
, r is a constant.
Find the conditions on the solution so that the sequence {u
n
} converges.
15
References
[1] Methods of Real Analysis, Chapter 2, R. Goldberg .
[2] Elementary Analysis: The Theory of Calculus, K. A. Ross.
[3] Calculus, G. B. Thomas and R. L. Finney, Pearson .
[4] Calculus, James Stewart, Brooks/Cole Cengage Learning.
16

Вам также может понравиться